Catalan et une Ramanujan — Les-mathematiques.net The most powerful custom community solution in the world

Catalan et une Ramanujan

Modifié (January 2022) dans Analyse
Bonjour
Je me suis confronté dans un fil à deux questions que je ne sais pas résoudre pour le moment 
1- Montrer que $\ \displaystyle \lim_{n\to +\infty}\sum_{j=1}^n  \sum_{i=n+1}^{\infty} \dfrac{1}{i^2+j^2}=G$
2- Montrer cette Ramanujan $\ \displaystyle\sum_{n=1}^{\infty} \frac{1}{n(e^{2\pi n}-1)} = \log 2 + \frac{3}{4} \log \pi - \frac{\pi}{12} -\log \Gamma \left(\frac{1}{4} \right).$
Le 😄 Farceur


Réponses

  • Pour flatter la paresse des lectrices, voudrais-tu rappeler qui est la constante de Catalan stp ?
  • Modifié (January 2022)
    Bonjour gebrane ceci me fait penser à cette formule: $\log \Gamma (z) = (z - 1/2) \log z -z + \frac{1}{2}\log (2\pi) + 2 \displaystyle\int_0^{+\infty} \dfrac{\arctan(t/z)}{e^{2\pi t} - 1 } \mathrm dt $.
    A demon  wind propelled me east of the sun
  • Modifié (January 2022)
    Bonjour
    Pour les tout aussi paresseux lecteurs: https://fr.wikipedia.org/wiki/Constante_de_Catalan
    Cordialement,
    Rescassol
  • Modifié (January 2022)
    Pour la 1 je ne vois pas encore à quoi ça peut servir la formule $\sum_{k=1}^{\infty}\frac{1}{k^{2}+j^{2}}=\frac{1}{2j^{2}}\left(j\pi\coth\left(j\pi\right)-1\right)$ mais elle est là !
  • La 1 n'est pas difficile : par comparaison à une intégrale on est ramené à calculer la limite de $\dfrac1n\displaystyle\sum_{j=1}^n\dfrac nj\arctan(\dfrac jn)$ qui est une somme de Riemann.
  • Modifié (January 2022)
    Dans le  message de @gilles benson on remplace la série par une intégrale, peut-on faire de même pour la 1 ?
    y a-t-il un lien entre $\ \displaystyle \lim_{n\to +\infty}\sum_{j=1}^n  \sum_{i=n+1}^{\infty} \dfrac{1}{i^2+j^2}$ et $\ \displaystyle \lim_{n\to +\infty}\int_1^n\int_{n+1}^{+\infty} \frac 1{x^2+y^2}dx dy$
    edit je viens de prendre connaissance du message de @jandri.
    Le 😄 Farceur


  • Bonjour
    Pour le 1)  est-il possible de trouver le  du second terme? C'est à dire que la somme s'écrit (peut être) $s_n=G+a/n +o(1/n)$  et que vaut $a?$ 
     
     
  • Un calcul numérique approché avec Maple me conduit à $s_n=G-\dfrac1{2n}+\dfrac5{24n^2}-\dfrac1{24n^3}+o(1/n^3)$ mais le dernier terme n'est pas bien étayé.
  • Modifié (January 2022)
    merci @Jandri Si je comprends bien tu fais  $n(s_n-G )$  et tu vois que cela s'approche  de  -1/2....  Le  développement est donc une conjecture ?
     
  • Exactement, ce n'est qu'une conjecture que je n'ai pas (encore) démontrée.
  • Modifié (January 2022)
    @jandri peux-tu détailler le 1) car j'ai fait autrement
     J 'ai reçu une réponse de la question de @bd2017 sut ME
    Le 😄 Farceur


  • gebrane

    j'ai écrit $\displaystyle\int_{n+1}^{+\infty}\dfrac{1}{t^2+j^2}dt\le\displaystyle\sum_{i=n+1}^{\infty} \dfrac{1}{i^2+j^2}\le \displaystyle\int_n^{+\infty}\dfrac{1}{t^2+j^2}dt$ d'où $\displaystyle\sum_{j=1}^{n} \dfrac{1}j\arctan(j/(n+1))\le s_n \le\displaystyle\sum_{j=1}^{n} \dfrac{1}j\arctan(j/n)$
    en utilisant $\arctan(x)+\arctan(1/x)=\pi/2$ quand $x>0$.

    Puis on passe à la limite dans les sommes de Riemann.
  • ok merci
    Le 😄 Farceur


  • Modifié (January 2022)
    Pour le 1) j'ai le développement suivant obtenu par calculs ... 
    $$s_n=G -\frac{1}{2 n}+\frac{5}{24 n^2}-\frac{1}{24 n^3}-\frac{1}{48 n^4}+\frac{1}{60 n^5}+\frac{43}{6048 n^6}+o(1/n^6)$$
     
  • Modifié (January 2022)
    Bonjour @bd2017 Peux-tu partager cette trouvaille avec tes lecteurs (sans laisser à leurs charges des points à vérifier :mrgreen: )
    Le 😄 Farceur


  • Bonjour
    C'est un peu long:  je donne l'idée.   J'établis  la relation : 
     $\sum_{p=n}^\infty  \dfrac{1}{p^2+j^2} =  \int_n^\infty  \dfrac{1}{j^2+x^2}  dx  +  \sum_{k\geq }  a_k  f_x^{(k)}(n,j)  +  ....$
    avec $a_k=-\frac{1}{2},\frac{1}{12},0,-\frac{1}{720},0,\frac{1}{30240},k=1,...$
    où j'ai posé $f(x,y)=\dfrac{1}{x^2+y^2}$  et $f_x^{(k)}$ et la dérivée  partielle de $f$  par rapport à $x$ $k$ fois.  Les pointillés  correspondent à un
    $o(1/n^m)$ (justifié)  et  $m$ étant la précision du développement que je souhaite. 

    Ainsi on   a $s_{n-1}$   la somme de $j=1$  à $n-1$  de l'expression précédente  qui sont (presque, il faut ajouter un terme  )  des sommes de Riemann des fonctions  $f_x^{(k)}(1,j)$ qu'on intègre sur $[0,1].$  Il reste à faire les développement de chaque  somme de  Riemann jusque la précision souhaitée. 
    Le DL  obtenu est donc justifié car je maitrise les termes négligés. 

    Par contre il y a moyen de vérifier  le résultat avec  un logiciel de calcul formel.  En effet on calculê  $s_{n+1}-s_n$ et le logiciel sait calculer sont DL.  Ce qui me permet de vérifier le terme en $1/n^6$ ci-dessus car j'avais un doute.  



     
  • Modifié (January 2022)
    Bonjour @bd2017.  Merci pour ta réponse.
    Elle n'est pas claire pour moi , peut-être c'est clair pour @jandri .
    J'ai espéré voir une preuve claire au moins de $s_n=G-1/2n +o(1/n)$, comment trouve-t-on rigoureusement ce -1/2, c'est ma question.
    Merci.
    Le 😄 Farceur


  • Modifié (January 2022)
    Ok tu as raison.  Obtenir le premier terme suffit à comprendre  comment on les obtient tous.  je garde les mêmes notation que ci-dessus
    1.pour tout  $p,  \int_p^{p+1}   f(x,j) dx = f(p,j)+\frac{1}{2}  f_x^{(1)}(p,j)  + O(\frac{1}{n^4}) $
    Cela s'obtient par Taylor au point $p$  et il faut comprendre que $O(\frac{1}{n^4})$ signifie que ce terme en valeur absolue est majoré par $\frac{C}{p^4}$
    la constante C ne dépend pas de $j\geq 1$.
    De même on obtient    $\int_p^{p+1}   f_x^{(1)}(p,j) dx =  f_x^{(1)}(p,j)  + O(\frac{1}{n^4}) $
    Donc  par élimination  $f(p,j)=\int_p^{p+1}   f(x,j) dx -1/2  \int_p^{p+1}   f_x^{(1)}(p,j) dx  +O(\frac{1}{n^4})$
    En sommant de n  à l'infini on a alors     $$  \sum_{p=n}^\infty  f(p,j)= \int_p^{\infty }   f(x,j) dx +1/2 f(n,j) +  O(1/n^3)$$2. On somme de j=1 à n-1   pour avoir:  
     $ s_{n-1} = 1/n \sum_{j=1}^{n-1}  \frac{\arctan(j/n)}{j/n} +  \frac{1}{2} \frac{1}{n^2}   \sum_{j=1}^{n-1}   f(1,j/n) +    O(1/n^2) $
    Il  faut ajouter le terme $j=n$  pour faire appraitre  les sommes de Riemann des fonctions     $\frac{\arctan(y)}{y}, f(1,y), f_x^{(1)} (1,y),...  $
    Pour chaque somme de Riemann il faut faire le développement asymptotique ....
    $s_{n-1}= [(G - 1/n) -\dfrac{1}{2 n}  \frac{1}{4} (\pi -4)]  + [\frac{1}{2} \frac{1}{n}  ( \int_0^1  f(1, y ) dy - f(1,1)/n )]  + O(1/n^2)  $
    $s_{n-1} = G -\dfrac{1}{2n}  +  o(1/n) $    
    P.S.  Remarque. Dans chaque intégrale qui intervient dans  le calcul  il y a un terme rationnel et un terme irrationnel.
     
  • Modifié (January 2022)
    Dans le même style que vaut $$\sum_{n=1}^{\infty}(-1)^{n}\left(\log2-\frac{1}{n+1}-\frac{1}{n+2}-\cdots-\frac{1}{2n}\right)^{2}$$
  • Modifié (January 2022)
    Merci@bd2017 pour les détails.
    @Boécien  je ne sais pas si ton exercice est facile ou non.
    Le 😄 Farceur


  • @gebrane Vu le document où je l'ai trouvé je mettrai comme niveau de difficulté *** (y a de méchantes intégrales doubles)
  • @Boecien: il y a un fil où on avait discuté de séries bâties sur les "queues" de séries bien connues mais je n'ai pas de lien. Les queues de séries telles que celle de $\ln(1+x)$ sont exprimables par une intégrale simple. La présence du carré laisse présager, en effet, le calcul d'une intégrale double.
  • Modifié (January 2022)
    @bd2017 le plus simple était de faire un Dl de $f$ à l'ordre 1 en tant que fonction à deux variables. Je peux rédiger avec détails si quelqu'un {en] fait la demande.
    L'exercice de @boécien me résiste encore. Normalement $\sum_{n=1}^{\infty}(-1)^{n}\left(\log2-\frac{1}{n+1}-\frac{1}{n+2}-\cdots-\frac{1}{2n}\right)^{2}=\sum_{n=1}^{\infty}(-1)^{n}\left(\gamma_{2n}-\gamma_n \right)^2$, il me faut une formule intégrale de $\gamma_{2n}-\gamma_n$, après voir $(\gamma_{2n}-\gamma_n )^2$ comme intégrale double.
    Le 😄 Farceur


  • Modifié (January 2022)
    L'exercice proposé par  Boécien ressemble à un exercice proposé par Chaurien en août 2021 dans le fil Séries et le nombre e.
    La somme à calculer peut s'écrire $S=\displaystyle\sum_{n=1}^{+\infty}(-1)^nu_n^2$ où $u_n=\ln2-\displaystyle\sum_{k=1}^{2n}\dfrac{(-1)^{k-1}}k=\int_0^1\dfrac{t^{2n}}{1+t}dt$.
    On en déduit $S=\displaystyle\iint_{0\le t,u\le1}\dfrac{-t^2u^2}{(1+t)(1+u)(1+t^2u^2)}dt\;du$ , intégrale qui n'est pas immédiate à calculer.
  • Modifié (January 2022)
    Bien vu jandri il fallait voir comment écrire cela en somme de "restes" comme l'énigmatique Fdp l'avait pressenti. Pour le calcul numérique avec pari-gp je viens de découvrir sumalt qui donne assez de précision dans ce cas
    u(n)=log(2)-sum(k=n,2*n,1/k)
    sumalt(n=1,(-1)^n*u(n)^2)
    -0.5420161222806063911636047550131098239
    pour que le détecteur de dépendance trouve immédiatement la formule qui, on pouvait s'en douter, fait intervenir les constantes de poids 2 G,Pi^2,Pi*log(2) et log(2)^2.
    Pour le calcul de cette intégrale double c'est du sport !
  • Modifié (February 2022)
    Grâce à l'alternance on peut fournir quelque chose de plus joli pour cette dernière somme en enlevant le log2 à gauche.
    \sum_{k=1}^{\infty}(-1)^{k-1}\left(\frac{1}{k}+\frac{1}{k+1}+...+\frac{1}{2k}\right)^{2}=\frac{5}{8}\zeta(2)-\frac{1}{2}G+\frac{3}{8}\pi\log(2)+\frac{3}{8}\log(2)^{2}

    edit: c'est faux, idem remarque ci-dessous

    @gebrane : désolé pour cette pollution de ton fil :/
  • Modifié (February 2022)
    En repassant par les poids 1 je trouve celle-là assez sympa
    \sum_{k=1}^{\infty}(-1)^{k-1}\left(\frac{1}{k}+\frac{1}{k+1}+...+\frac{1}{3k}\right)=\frac{\pi}{3\sqrt{3}}+\frac{2\log2}{3}
    Edit:  résultat faux, cf messages suivants
  • Bonjour
     Je ne suis  pas convaincu de ton dernier résultat. Les somme partielles sont bornées mais rien ne dit que la série converge.
    Il me semble que tu as formellement  interverti somme  et intégrale  et alors    peut être que ton résultat concerne la limite des somme d'ordres partielles paires (ou bien impair).  De toute façon la différence de 2 sommes partielles consécutives   s'approche de $\pm log(3)$ 
     
  • Oui c'est faux les sous suites S(2n) et S(2n+1) convergent vers je ne sais quoi.
  • Modifié (February 2022)
    Non  tu peux laisser.  Si on désigne par L  ta limite,  l'une des 2 (je ne sais plus laquelle)  tend vers L   et l'autre vers $L\pm log(3)$    (log(3) ) étant la limite du terme général en valeur absolue.
     
  • La série proposée par Boécien, $\displaystyle\sum_{n=1}^{\infty}(-1)^{n}\left(\log2-\frac{1}{n+1}-\frac{1}{n+2}-\cdots-\frac{1}{2n}\right)^{2}$, conduisant à une intégrale trop compliquée à calculer je propose une série très proche qui conduit à une intégrale qui se calcule très bien : 
    $S=\displaystyle\sum_{n=0}^{+\infty}(-1)^n\left(\log2-\sum_{k=\lfloor n/2\rfloor+1}^n\frac1k\right)^2$ (avec la convention $\displaystyle\sum_{k=1}^0u_k=0$).

  • Modifié (February 2022)
    Effectivement un petit changement et les difficultés s'envolent !
    $$\sum_{n\geq1}(-1)^{n-1}\left(\log2-\sum_{k=\left\lfloor n/2\right\rfloor +1}^{n}\frac{1}{k}\right)^{2}=\sum_{n\geq1}(-1)^{n-1}\left(\sum_{k\geq n+1}\frac{(-1)^{k-1}}{k}\right)^{2}=\log(2)^{2}-\frac{\zeta(2)}{4}$$
  • Exactement. En ajoutant le terme avec $n=0$ le $(\log 2)^2$ disparait.
  • Unexemple avec les sommes de Cesàro  ($M_n(u)=\frac 1n\sum_{k=1}^n u_k$).
    Montrer que$\sum\limits_{n = 1}^\infty (-1)^{n-1}M_n(\frac 1n)M_n(\frac{(-1)^{n-1}}{n^2} )= - \frac{{13}}{{48}}{\pi ^2}\zeta \left( 3 \right) + \frac{{125}}{{32}}\zeta \left( 5 \right)$

    Le 😄 Farceur


  • Modifié (February 2022)
    Sinon pour revenir à la question de gebrane sur la "Ramanujan", on peut obtenir une intégrale complexe pour la série en développant en série la fraction et en utilisant une représentation intégrale du polylogarithme ici. Sauf erreur
    $$\sum_{n=1}^{\infty}\frac{1}{n\left(e^{2\pi n}-1\right)}=\frac{1}{2\pi i}\int_{c-i\infty}^{c+i\infty}\frac{\Gamma(z)}{\left(2\pi\right)^{z}}\zeta\left(z+1\right)\zeta\left(z\right)dz.$$
    Mais je ne sais pas la calculer.
  • Bonjour   En transformant  lé série en produit puis avec wolfram on a  
    $$S=\sum_{n=1}^{\infty}\frac{1}{n\left(e^{2\pi n}-1\right)}=\sum_{n=1}^{\infty}\frac{1}{ne^{2\pi n} } \sum_{k\geq 0} exp(-2\pi n k) =-\sum_{k\geq 1}log   (1 -exp(-2 \pi k)) $$

     $S=-\frac{\pi }{12}-\text{Log}\left[\frac{\text{Gamma}\left[\frac{1}{4}\right]}{2 \pi ^{3/4}}\right]$



     
  • Modifié (February 2022)
    Bonjour   En transformant  lé série en produit puis avec wolfram on a  
    $$S=\sum_{n=1}^{\infty}\frac{1}{n\left(e^{2\pi n}-1\right)}=\sum_{n=1}^{\infty}\frac{1}{ne^{2\pi n} } \sum_{k\geq 0} \exp(-2\pi n k) =-\sum_{k\geq 1}\log   (1 -\exp(-2 \pi k)) $$
     $S=-\dfrac{\pi }{12}-\log\bigg(\dfrac{\Gamma(\frac{1}{4})}{2 \pi ^{3/4}}\bigg)$.
     
  • Bonne idée @bd2017 , il suffit de démontrer ce que wolphi sait faire
    A méditer
    Le 😄 Farceur


Connectez-vous ou Inscrivez-vous pour répondre.
Success message!